Você está na página 1de 61
beau From the figure shown, the cross-sectional areas of the two wires are 400 mm? for wire ABand 200 mm? for wire AC. ® Compute the largest weight that can be supported by wire AG if the stress is not to exceed 100 MPa. ® Compute the largest weight that can be supported by wire AB if the | stress is not to exceed 150 MPa. ® Determine the largest safe weight. Solution: © Largest weight that can be supported by wire AC if the stress is not to exceed 100 MPa. Consider the FBD of joint,A with x-axis through ABas shown By=0 AC Sin 75° = W Sin 60° AC = 0.897 W SH= AB Cos 30° = AC Cos 45° = (0.897 W) Cos 45° AB = 0.732 W P=AS For AC: AC = (200)(150) AC = 30,000N 0.897 W = 30,000 W = 33,445 N @ Largest weight thatcan be supported by wire AB if the stress is not to exceed 150 MPa. AB = 400.(100) ‘AB = 40,000 N 0.732 W = 40,000 W = 54,645 N @ Largest safe weight. Largest safe weight W = 33,445 N For the truss shown, the cross-sectional area of each member is 1200 mm? © Compute the stress in member DE. ® Compute the stress in member CE, @ Compute the stress in member BD. Solution; © Stress in member DF. Consider FBD of whole truss 3M, =0 Rp (10) = 100 (4) +200 (7) Rp= 180 kN Consider FBD of joint F EVve0 4 5 DF 160 DF = 225 kN (C) Pass the cutting plane a-a through “es members BD, CD, and CE and consider the FBD of right segment. EF. fam IMp=0 . CE (4) = 1803) R= 180 CE=1354N (7) Resolve BD into its component at B TMc =0 3 == BD (6) + 200 (3) = 180 (6) aa (6) + 200 (3) (S) BD = 96.15 KN (C) ‘The stresses: Fr Sai For DF: Sor ARO 187.5 MPa (C) 200 KN @ Stress in member CE. zee Seg = 13310000) _ 125 Pa (1) @ Stress in member BD. San = 5 1000) «go mPa (0) Gea For the truss shown 4 reduced stress in compression is specified to avoid the danger of buckling. ® Determine the cross-sectional area of bars CF so that the stresses will not exceed 100 MN/m? in tension or 80 MN/m? in compression. ‘@ Determine the cross-sectional area of bars BE so that the stresses will not exceed 100’ MN/m? in tension ‘or 80 MN/m? in compression. ® Determine the cross-sectional area ‘of bars BF so that the stresses will not exceed 100 MN/m? in tension. or 80 MN /m? in compression, 4 (ii Solution: © Cross-sectional area of CF: Pass the cutting plane a - a through members BE, BE, and CF, and consider the FBD of the right segment EMp=0 CF (6) = 40 (3) + 50 (6) CF=525KN (C) Resolve BE into its component at joint E =M=0 $e (4) = 50 (3) BE=625KN (1) EV-0 8 4 Tee +50 BF = 42.72 kN (1) @ Cross-sectional area of BE; Age =625{2000 Ape = 625 mun? @ Cross-sectional area of BF: Awe AL om) = 427.2 mm? 10m © Determine the maximum load P that can be applied so that the stresses of bar AB will notexceed 100 MN/m? in tension or 80 MN /m? in compression. @ Determine the maximum load F that can be applied so that the stresses of bar BC will not exceed 100 MN/m? in tension or 80 MN/‘m? in compression. ‘® Determine the maximum load P that can be applied so that the stresses of bar AC will not exceed 100 MN/m? in tension or 80 MN/'m? in compression. Solution: rf © Maximum load P for bar AB: Consider the FBD and force polygon for joint B BC 4Peos0=08P (C) Fs < AB=Psin6=06P (C) AB BC Consider FBD of joint A AC =(Q6P) cos 0 AC = (0.6 P)(0.8) he AC =048P (T} Be Peas For AB: 0.6 P = (80 x 60)(80) AB P= 240,000 N @ Maximum load P for bar BC: 0.8 P = (20 x 60)(80) ‘AB=0.6P P = 180,000 8 @ Maximum load P forbar AC: o 0.48 P = (30 x 60)(100) P = 375,000.N MA 4 cast-iron eolumn supports an axial compressive load of 250 KN. Determine the inside diameter of the column if its outside diameter is 200 mm and the limiting, compressive stress is 50 MPa. Solution: A=fO2- Dy} A=F (40,000 - D,?) PAS 250 (1000) = F (40,000 - B,2)450) 5000 = F (40,000 - D,2) 40,000 - Dy? = 6366.2 Dy = 183.4 mo Determine the outside diameter of a hallow steel tube that will carry a tensile load of 500 KN at a stress of 140 MPa. Assume the wall thickness to be one-tenth of the outside diameter. Solution: P S=5 A=35714 mm? A=F [D? - (080) A=FID?- 0.64 D4) 3571.4 = 7 (036 DY) Ds 1124 mom [SIMPLE STHEES} ee A hollow steel tube with an inside diam, of 100. mm must carry an tensile load of 400 KN. Determine the outside diameter of the tube if the stress is limited to 120 MPa. Solution: o= Pho 400000 10= gD (100)7) D? ~ (1007 = 4244 D= 119.3 mm, a A homogeneous 800 kg bar AB is supported at either end by a cable as shown in the © Compute thel&atio of the strength of steel to that of bronze. ® Determine the smallest area of steel cable it the stress is not to exceed 120 MPa. ® Determine the smallest area of the bronze cable if the stress is not to exceed 90 MPa. Solution: @ Ratio of the strength of steel to bronze, EM, =0 10 Ty = 7849(5) T, =382N Bo0(9 t)eTEA NS Eh=0 Ty, + Tp = 7848 Ty = 748 - 3924 = 3924 3924 Ratio =3924- 10 @ Area of steel cable. eee a, 120 Ag = 327 mm? ® Area of the bronze cable. yeh 4 5 90 Ay = 43.6 mn? Ede ‘The homogeneous bar shown is supported by a smooth pin at C and cable that runs from A to B around the smooth peg at ID. The bar weighs 6 KN. © Find the tension on the cable. ® Find the reaction atc. ® Find the stress in the cable if its diameter is 15 mm. Solution: @ Tension on the cable. tan 6 = 8 = 30.96" EMc = 0: T Sin 30.96°(10) + T(5) = 6(5) T=2.96kN T=2960N @ Reaction at C. =F, 20 TSin 20.96" +T +C, =6 2.96 Sin 30.96" + 2.96+ Cy =6 C, = 1.52 kN EF, =0 C, = T Cos 30.96" C, = 2.96 Cos 30.96" C, = 254 kN C= Vi2.54)? + (1.52)? = 2.96 KN ® Stress in the cable, oa, = 2960 A Bas gus o= 16,75 MPa For the truss shown: ® Calculate the stress of member CE. ® Calculate the stress of member DE. @ Calculate the stress of member DF. ‘Assume cross sectional area of each member is 1.8in*, Solution: @ Gress of member CE. IM, =0. 24 Ry = 3016) Ry = 20kips. Rp = 30-20 R= 10 kips. EMp=0 10(8) = CE Cos 6 (6) 10(6) = S28) (6) CE = 16.67 kips 10 [Simrie Sars) 16670 Sees ice = 9260 poi, (Tension) @® Stress of member DE. EMg=0 20(8) = DB Cos @ (6) 2046) = PELE) 6) DB = 33.33 D ba Dras.33 DE = 40 Kips. tite DE = 33.33 Sin. (2) pe =32358) a 40000 ODE 38 Ope = 22222 psi Opp = 22.2 ksi (Tension) @ Stress of member DF. 23333 Spr =e ‘Opp = 185 ksi. (Compression) Ir Ue Mace ‘. From the truss shown. ® Determine the cross sectional area of member AG if the stresses should not exceed 20 ksi in tension and 14 ksi in compression. A reduced stress in compression is specified to reduce the danger of buckling. @ Determine the cross sectional area of member BC. ® Determine the cross sectional area oftmember CE. Solution: ‘D Cross sectional area of member AG. EM, =0 3(6) Ry = 40(4) + 25(8) ' Ry = 20kips. SF, =0 Ry =R, = 20 kips. tRs0 Ry = 40 +25 Ry = 65 kips. 6 tan 8=7 0 563" EF, =0 AB Sin @ = 65 ay AB Sin 56.3" = 65 AB = 78.13 kips. 20 e ER-0 AG AG +20= ABCos@ é AG + 20» 78.13 Cos 56.3" AG = 2335 kips (7) Agg = LIT int (Cross sectional area of member BC. TMp=0 BC Sin 0 (4) + 40(4) + 20(6) = 65¢8) BC Sin 56.3" (4) = 240 BC=7212kips (Comp) Cross sectional area of member CE. CD Cos 56.3" = 20 CD = 36.05 kips DE =CDSing DE = 30 kips EF Sin 56.3" = 30 EF = 36.05 kips CE =EFCos8 CE = 36.05 Cos 56.3" P ony 20 4 Ace = 1.43 in? The homogeneous bar ABCD shown in the figure is supported by a cable thal runs from A to B around the smooth peg at E, a vertical cable at C and a smooth inclined surface at D. Determine the mass of the heaviest bar that can be supported if the stress in each cable is limited to 100 MPa. The area of the cable AB is 250 mm? and that of the cable at Cis 300 mm?, Solution: T, = 100(250) T, = 25000 N Tz = 100(300) T; = 30000 N IMp=0 T; Sin 20° (6) +T; (4) + T (2)=3W r, 25000 Sin 30° (6) + 25000(4) + 30000(2) = 3W W = 7833333 N REL: Part of the landing gear for a light plane is shown in the figure. Determine the compressive stress in the strut AB caused by a landing reaction R = 20 KN. Strut AB is inclined at $3.1" with BC. Neglect weights of the members. id (Gan) Solution: EMc = 0 «P Sin 53.1)(450) = 20 (650) P P= 36.125 kN a A=Fldor- 07] = 5498 mm? i = BOD 65.71 MPa CI A steel tube is rigidly attached between an aluminum rod and a bronze rod as shown inthe figure, Axial loads are applied at the positions indicated Res wow ie ® Find the maximum value of P that will mot exceed a stress in aluminum of 80 MPa. @ Find the maximum value of P that will not exceed a stress in steel of 150 MPa. ® Find the maximum value of F that will not exceed a stress in bronze of 100 MPa. Ateminen Sicel Bice 30 Soiution: Maximum value of P that will not exceed a stress in aluminum of S0MPa. e F 80= 505 P= 16,000 N @ Maximum value of P that will not exceed a stress in stewl of 150 MPa 3 Maximam value of P that will not exceed a stress in Bronce of 100 MP 4 100 = 00 4 P= 12,500N eciial From the figure shown, @ Compute the maximum value of P that will not exceed a stress of bronze of 100 MPa. @ Compute the max, value of P that will not exceed a stress of aluminum of 90 MPa. @ Compute the maximum value of F that will not exceed a stress of steel of 140 MPa. eae Brense Aluminum Stet eco aral Gen hoa Solution: Max. value of P that will mot exceed stress of bronze (100 MPa), ; fe bre ° ao Pear 3P 100= G50 P= 15,000 PEPE br Perskw See in 80 e ® Max. value of P that will not exceed stress of aluminum (90 MPa) 5-2 A 90+ 35 P=270N Ps27kN ® Max. value of P that will not exceed a steel stress of 140 MPa. ast a qin Te M0 = 355 P= Z1Q00N P=21kN AnD pent AmGOO ment lOO een? A rod is composed of an aluminum section rigidly attached between steel and bronze sections. Axial loads are applied at the positions indicated. If P = 3000 lb. and the cross sectional area of the rod is 0.5 in? © Determine the stress in steel. ® Determine the stress in bronze. i pee Bre @ Determine the stress in aluminum = Pi Ey En 2S Solution: @ Stress in steel. a TE ee oy ae 3 FE 42 ap vd %=55 ET (3000) apr o.=G5 . aes 6, = 24000 psi = 24 ksi © Stress in bronze. 3P bap oS ae Op = 18000 psi = 18 ksi ® Stress in aluminum, 4 oanqe @, = 4(3000) OS 3, = 24000 psi = 24 kat Cedi An aluminum rod is rigidly attached between a steel rod as shown. Axial loads are applied at the positions indicated. Find the maximum value of P that will not exceed a stress in steel of 140 MPa, in aluminum of 90 MPa or in bronze ef 100 MPa. Solution: ‘Safe value of P= 10kN Ls Fo Breme oP P= 10000 N ees ‘The bar supported by a pin at A and a, cable at B carries a load of 260 N at C. Neglecting the weight of the bar. ® Which of the following gives the normal force on the bar segment on the left of a section which is 3 meters from A. ® Which of the following gives the shear force on the bar segment on the feft of 0 section which is 3m. from A. ® Which of the following gives the bending moment on the bar segenent on the left of a section which is3 m. from A. 18 (esate) Solution: ® Normal force 3m. from A. IM, =0 T'Sin 30°10) = 260(42)(6) T = 288 BF =0 Ray + 288 Sin 60" 20002) 13 Ra =-942 (downward) =Fh=0 Rx = 288 Cos 60°-260 (3) Rax=44 Normal force = 44 @ Shear force at 3 m. from A V=94d2N ® Bending moment at 3m. from A. M = 9.42(3) M = 28,26 Nan fs] Freon A homogeneous 150 kg bar AB carries a 2 kN force as shown in the figure.’ The bar is supported by a pin at B and a 10mm diameter cable CD. ® Find the tension in the cable CD, ® Find the stress in the cable in MPa. @ Find the reaction at B. Solution: @ Tension in the cable CD, 19 EM,=0 $1) = 1.4709) + 216) T= 6.839 kN ® Stress in the cable. P Sak § =$:832(1000) _ 57.08 mPa zaoyr 2kN WeL47kN @ Reaction at B et(* B,+2+147=T(#) B, +347 = 6.839 ® By =< 2kN Bam 6aae(> =410kN Re= Voy sao Ra=456kN A weight W is suspended from a fine wire AB and’a very flexible wire BCD which passes over a frictionless pulley at C. The end of the wire BCD is attached toa 10: kN. weight, and the wires makes an angle shown with the vertical. © Which of the following gives the tension in wire AB, ® Which of the following gives the weight W. ® Which of the following gives the vertical reaction at C. Solution: ® Tension in wire AB: TAB Sin 45° = 10 Sin 60° TAB = 12.2 kN ® Weight W: W = 12.2 Cos 45" + 10 Cos 60° W=13.6KN @ Reaction at C: Re=10+10Sin30°=15 kN perio Determine the weight of the heaviest cylinder which ean be placed in the position shown in the figure without exceeding a stress of 50 MN/mt? in the cable BC. Neglect the weight of bar AB, The cross-sectional area of the cable BC is 100 mm?. Solution: T = 50(100) T= 5000 N EM,=0 Péd) = 5000 (8) P= 10,000N Consider FBO of cylinder WePcosé W = 10,000 ®) W = 6,000 N the weight of heaviest cylinder Pata A rod is connected to a pin at A and a chord at Bas shown. Itholds a cylindrical dram which weighs 176 N. The drum has a diameter of 2m. © Which of the following gives the force between the drum and the rod. @ Which of the following gives the force in the chord BC. @ Which of the following gives the reaction at the pin at A. Solution: @ Force between the drum and the rod. DF=0 Ro Cos 60" = 176 Rp =352N ® Force in chord BC. map tan 15" = 25 Sone tanO=7 0= 36.87 aria 3687 2Ma=0 352(3.732) = T Cos 6.87°(10) T= 13232 @ Reaction at pin A. EF, =0 Ay + 132.32 Sin 36.87" = 352 Sin 20° Ay =96.61N EF, =0 ‘Ay + 132.32 Cos 36.87" = 352 Cos 30° An= 198.94 N Ra = V (i994)? + (96.61)2 Ra =221.19N A 1000 kg homogencous bar AB is suspended from two cables AC and BD; each wit! cross-sectional area 400 mm, as shown in the figure. The stresses in the cables AC ar BD are limited to 100 MPa and 50 MPa, respectively. © Compute the tension force of bar BD. ® Compute the magnitudeof P. ® Compute the location of P from A. Solution: ‘© Tension force of bar BD. Peas T, = (400}(100) T, = 40,000 N Ty = (400)(50) Tp= 20,000 N @ Magnitude of F. W = (1000)(9.81), W = 9310N SFy=0 P+Waety+Ty P = 40,000 + 20,000 = 9810 P= 50,190 N ® Location of P from A. EM, =0. 50,190) + 9810(1) = 20,000(2) x = 6.602 m, 23 ‘The lower ends of the three bars in the figure are at the same level before the rigid homogeneous 18 Mg block is attached. Each steel bar has an area of 600 mm?and E = 200 GPa. For the bronze bar, the area is 00:mum? and E = 83 GPa. zie) © Which of the following gives the stress developed in the bronze bar. @ Which of the following gives the stress developed in the steel bar. ® Which of the following gives the elongation of the bars. Solution: ® Stress developed in the bronze bar. Pp + 28, = 18000(9.81) Asip Sp +25, .Ag = 18020(9.81) 900(Sp) + 2(3.8555,X600) = 18000(9-B1) Sp = 51.95 MPa @ Stress developed in the steel bar. 55 = 3.855 Sp Sq = 3.855(31.95) §,= 123.2 MPa 18,000 kg ® Elongation of the bars are equal. 5 = _.123.2(1000) 5s = 200000 8, = 0.62 min ‘A rigid block weighing 176.4 kN is attached as shown on the figure: If the block is to remain horizontal, ® Which of the following gives the tatio of the load carried by each load. @ Which of the following gives the stress of the bronze. ‘@ Which of the following gives the elongation of the rods. Solution: {@ Ratio of load carried by each rod. a, = 8, Pols . Poly (AE), (AE), PsQ) Py (1.6) 600 (200) = Py = 2.57 Py ite 257 © Stress of Bronze. 2P,+P,= 1764 2(2.57P,) + Py = 1764 Py = 28-73 kN 6, = 287341000 Sp = 31.92 MPa @ Elongation of the rods. PL =a *o* TAR), L ge ale Sue 31.92(1.6)(1000) e 83000 5, = 0.615 mm EZ ‘A.uniform concrete slab of mass M is to be attached as shown in the figure to.two rods whose lower ends are initially at the same level. © Which of the following gives the area of aluminum bar so that the slab will remain level after it is attached to the rods. 4 @ Which of the following gives the stress in the steel rod if M = 18000 kg ® Which of the following gives the stress in the aluminum rod if M = 18000 kg. Solution: ® Area of aluminum bar. Bie eg, (AE (AB, EM, =9 Fa(5) = WO) Pi=EW P,+F,=W Ppa EW 2/5 W(3) _ 3/5 Wi6) A000) = ACD) A = 257% mm? ® Stress of steel bar. 2 Pe=2W =e (18000)(9.81) Py = 70632 Py _ 70632" cee Ss" ao""300- 5, = 235.44 MPa ® Stress of aluminum bar. P, =2 (18000)(9.81) = 105048 N P, _ 105948 S.A, oar S_= 41.20 MPa applied. Solution: ® Elongation of steel. 900070) ~ 600(200) Pa=14Ps BPs + 6(14) Py = 12005) Py = 52.63 kN 's 4(52.63) Py = 73.68 kN a= e 5, = 3.51 mm e Vertical movement of P. ae no SUL75) 2.97 mm bp = Eacaae As shown in the figure, a rigid beam with negligible mass is pinned at one end and supported by two rods. The beam was initially horizontal before the lead P was D Compute the elongation of steel if P = 120 kN. ® Compute the elongation of aluminum ifP = 120 kN. ® Find the vertical movement of Pif P= 120 kN. a... 73680(3000) Eq —900(70000) As in the figure shown, a hole is to be punched out of a plate having an ultimate shearing stress of 300 MPa @ If the compressive stress in the punch is limited to 400 MPa, Determine the maximum thickness of plate from which a hole 100 mm ‘in diameter can be punched. @ Ifthe plate is 100 mm thick, compute the smallest diameter hole which can be punched, Solution: . Maximum thickness of plate. PeAS =} (100)? (400) = 1,000,000 2 N a From shearing of plate, Compan rs Ay=xDt=100Kt Sheard area + ae S ne SS 1,000,000 r = (100% 1)(300) =a $= 33.33 mm @ Smallest diameter hole. Pras, Pa(eDt)Ss P =5D (10}(300) 000 & D From compression of punch PAS P=7D* (400) Pa 1002 D 100 x D?= 3000. x D D= 2.0mm 28 Problent The end chord of a timber truss is framed into the bottom chord as shown in the figure. Neglecting friction, @® Compute dimension b if the allowable shearing stress is 900 kPa. @® Determine dimension C so that the bearing stress does not exceed 7 MPa. Solution: © Dimension b if the allowable shearing stress is 900 kPa. Py= AS, P, =P cos 30" 50 Cos 30° (1000) = (150 b)(0.9) b= 320,75 mm, say 321 mn ® Dimension cso that the bearing stress does not exceed 7 MPa. Py= An Sp 50 Cos 30° (1000) = (150c)(7) ¢= 41.24 mum, say #2 nai en In the landing gear described as shown in the figure, the bolts at A and B are in single shear and the one at C is in double shear. ® Compute the required diameter of bolt A if the allowable shearing stress is 50 MPa. @- Compute the required diameter of bolt B if'the allowable shearing stress is 50 MPa. ® Compute the required diameter of bolt C if the allowable shearing stress is 50 MPa. 29 “Solution: © For bolts at A (single shear), EMe = 0 (P sin 53.1)(450) = 20(650) P= %4.125KN Pa AS Pp 1000(36.125) = (#p2) (50) 53.4" D = 30.33mm Ch @ For bolts at B (single shear). 200 kN cy Ps AS 1000(36.125) = ( #12) (50) D = 30.33mm For bolt at C (double shear). TV so Cv +20 = 36.125sin 531° Cv = BSB9KN 2H =0 Cy = 36.125 cos §3.1° CH = 21.690 kW R, = Veqt 4+ ey" R= Vai + aaa? R, = 2344kN = 23.44(1000) = (7 D%) (602) D = 173mm Solution: @ Max. flexural stress, A 750-mm pulley, loaded as shown, is keyed to shaft of S0-mm diameter, ® Determine the force acting on the ia 7Smmlong key. @ Determine the width b of the 7S-ma-long. key if the allowable EC shearing stress is 70 MPa. es f (10) (0.40) Solution: max 300 fmax = 266.67 MPa (©. Force acting on the 75 mm long key: aa EM, = 0 P(25) + 6(375) = 10(375) P= GOkN @ Min, diameter of pulleys. R moot? 409 9) = 202.00 1040) =200 1m @ Width "b" of the mm tong key. Paas T D=2R =2.(200) 60(1000) = (756) (70) | D = 400 min b = Lidmem w, 20 mim wide k 200 GPa, ¥y 0.80 mm thick, runs over pulleys swing most nearly gives the maximum flewural stress developed? = following most neatly gives the minimum diameter of pulleys wi 5 of 400. MPa? nearly gives the minimum thickness of the steel band saw without exceeding a flexural stress of 400 MPa? The bell crank shown in the equilibrium @ Determine the value of P for @ Determine the required diameter of the connecting rod AB if its axial stress is limited to 100 MN /m? ® Determine the shearing stress in the pin at Dif its diameter is 20mm. Solution: @ Value of P for equilibrium. =Mp = 0 200P = 30sin 60 (240) P = 32.177 kN ® Diameter of the connecting rod AB, P=ASs 31.177(1000) = (a? ) (100) d = 19.92 mm ® Shearing stress in the pin at D. =F, = 0 DH = 31177 +30c05 60° DH = 46.177 kN ER =0 Dy = 3sin 60° Dy = 25.980 kN Rp = Ved77e + 2598 Rp = 52.984 kN (52.984(1000) F (20)? @) 5S = 84.33 MPa Sia k (double shear) ae 33 eee ‘The mass of the homogeneous bar AB shown in the figure is 2000 kg. The bar is supported by a pin at B and a smooth vertical surface at A, © Compute the vertical reaction at B. ‘@ Compute the horizontal reaction at B, ® Determine the diameter of the smallest pin which car be used at B if its shear stress is limited to 60 MPa, Solution: © Vertical reaction at B. W = (2000)(9,81) = 19,620N ® Horizontal reaction at B. EMA = 0. By (8) + 19,620(3) = 19,620(6) By = 7357.5. @ Diameter of pin at B. Rp = ¥(7357.5)* + (19,620)? Rp = 20,954N P = AS (doubleshear) 20,954 = (F d?) (60y2) d= 149mm ‘Two block of wood, 50 mm wide and 50 mm thick, are glued together as shown in the figure. @ Using the free-body diagram concept, determine the shear load in the glued joint if P=AO00N. @ Compute the shearing stress in the glued joint. @ Generalize the procedure of part { to show that shearing stress on/a plane inclined atan angle 8 toa transverse section of area A is Ss =P sin2@ / 2A. Solution; ® Shearing load. V = Poos60" V = Peos 60" = 6000.cos 60" V = 3000N & seine ‘stress in the glued joint. Se 3 §, = 2.598 MPa @ .Shearing stress on a plane inclined at an angle © to a transverse section of area. V = Psin® Shearing stress, ¥ 5" 5 Psin = cos @ phe Psinboos® Sin20 a 2sin Bcos@ _ Psin te 2A A rectangular piece of wood, 50 mm by 100 mm in cross-section, is used asa compression block as shown in the figure. The grain makes an angle of 20° with the horizontal, as shown, @® Determine the maximum axial load fe F which can be safely applied to : the block if the compressive stress in the wood is limited to 20 MN/m?. @ Determine the maxittum axial load Powhich can be safely applied to the block if the shearing stress parallel to the grain is limited to = 5MN/m2. Solution: © Max. axial load P from compression P=Sa P = (20\(50)(100) P = 160,000. ‘@ Max. axial load P from shear, Psin28 a pa2AS sin 26 p = 21501000)5) sin 40 P = 7786N 36 In the figure shown, assume that 2 20-mm rivet joins the plates which are each 100 mm wide. @ Compute the value of P due to shear of rivets. Allowable shearing stress is 80 MPa. ® If the allowable stresses are 140 MN/m? for bearing in the plate material, determine the minimum thickness of each plate. @ Under the conditions specified in part (a), what is the largest average tensile ‘stress in the plates. Solution: @ Value of P due to the shear of rivets, Pe As, F =F (20)? (60) P = 25.133N @ Min, thickness of plates. From bearing of plate P= ApS, P = (dS, 25,133 = (208) (140) t = 8.98 mm © Tensile stross in the plate P s-f A = (100)(8.98) - (209(8.98) A= 718mm? 35) = 23133 Se. Fig = 35.00 MPa projected aren af rivet hoe ‘The lap joint shown in the figure is fastened by three 20 mm. diameter rivets. Assuming that P'=S0 kN, @ Determine the shearing stress in each rivet, @ Determine the bearing stress in. each plate. @ Determine the maximum average tensile stress in each plate. Assume that the axial load P is distributed equally among the three rivets. Solution: © Shearing stress in each rivet See AS = (50){1000) r Foy 3) S$, = 53.05 MPa ® Beating stress in each plate P S=a5 _ 50 (1000) $= BON) Sp = 33.33 MPa ® Maximum tensile stress in each plate Sa Anat Arner = (130 - 20)(25) Anet = 2750 mm? 50 (1000) . S=""3750 S= 18.18 MPa For the lap joint shown in figure. ‘©. Determine the maximum safe load P which may be applied if the shearing stress in the rivets is limited to 60 MPa, ‘® Determine the safe load P_ which may be applied if the bearing stress of the plate is limited to 110 MPa. @ Determine the safe load P if the average tensile stress of the plate is Limited to 140 MPa. Solution: ‘Safe load P due to shear of rivets. Pas P=] (20)? (60)(3) P= 56,549 N S ® Load P due to bearing of plates. P=AyS P = (20(25)(110)(3) P = 165,000 N ® Load P due to tearing of plates. PaAneS Ane: = (130 - 20)(25) Anet = 2750 mm? P= (2750)(140) P = 385,000N Therefore, maximum safe load P = 56,549 N (shearing of the rivets govern) In the elevis shown in the figure, it support a load P = 55 kN, © Determine the min. bold diameter “d" without exceeding the allowable shearing stress of 70 MPa. ® Determine the thickness of each yoke that will support the load P without exceeding the allowable bearing stress of 140 MPa. Solution: © Minimum diameter of bolt (double shear) P=AS 55 (1000) = (F #) (roy) d= 22.37 mm @ ‘Thickness of each yoke Pls Ay Sp ra Py =27.5kN (foreach yoke) Apadt=22371 27.5: (1000) = (22.37 (140) {8.78 erm en teh A certain engineering equipment is supported by the foundation shown. The diameter ‘of the circular pin is 25 mma. Fy = 345MPa, © Which of the following gives the vaue of P $0.45 nat to exceed the allowable bearing tress of the circular pin. @ Which of the following gives the value of P so as not to exceed the allowable shearing stress of the circular pin. @ Which of the following gives the value of P so as not to exceed the allowable hending stress of the circular pin. Solution; ©. Value of F so as not to exceed the allowable bearing stress, Allowable bearing stress = 0.90Fy Fa ays, F = 25(50)(0.90)(345) P= 776250 N P= 776.25 kN | ® Value of P so as not to exceed the allowable shearing stress. Allowable shearing stress = 0.40Fy FA,5, (Double shear) BaF (25 7(0.401045)2) P = 270962 N = 271 KN Single shear for outer member $5 aE: Be © ios? 7 = 040(345) F (25) P=67741N P= 67.74kN Safe P = 67.74 kN ws @ Value of P soas no to exceed the allowable bending stress. Allowable bending stress = 0.66Fy M=$(62)=31P = Mc Rey o.6s(245) = M023 aay M = 349287 349287 = 31 P P=11267N P=11.27kN 422.2 mm diameter bolt having a diameter at the root of the threads of 18.6 mmis used to fasten two timbers as shown in the figure. The nut is tightened to cause a tensile load in the bolt of 34 KN. @ Determine the shearing stress in the head of the bolt. ® Determine the shearing stress in the threads. ® Determine the outside diameter of the washers if their inside diameter is 28 mm and the bearing stress is limited to 6 MPa. Solution: © Shearing stress in the bead of the bolt $= As s, = 34(2000) mm (22-2)(12) 8, = 40.625 MPa @ Shearing stress in the threads 5, = 24 (1000) “a (86)16) S, = 36.366 MPa ® Outside diameter of washer Po= ApS, 34 (1000) = Fle? - (287) (6) Fld? - (26)"] = 5666.67 d= 89.44 mm ‘The figure shows a roof truss and the detail of the riveted connection at joint B. Usi allowable stresses of S, = 70 MPa and S, = 140 MPa, Area of 75 x 75 x 6 = 864 mmé, Areaof 75x 75x 13 = 1780 mm’, ® How many 19 mm diameter rivets are required to fasten member BC to ‘the gusset plate? @ How many 19 mm diameter rivets are required to fasten member BE to the gusset plate? @ Determine the largest average tensile stress in member BC. ® Determine the largest average compressive stress in member BE. Solution: ® No, of 19mm rivets for member BC. Due to shear of rivets: From jointC: Spc = 96 kN P=A,Sn 96000 = F (19)7(70){n) n=484 Due to bearing of rivets: Pe Ayn 96000 = 19(6)(140)(n) n= 6.02 say 7 rivets Usen = 7 rivets for member BC @ No. of 19mm rivets for member BE. EM, =0 2 BE (8) = 95.4) BE=80kN (compression) aC Due to shear of rivets: P=AsSn AC: cE x 000.= 5 (19)? (70){n) sek n= 403 say 5 Due to bearing of rivets. 80,000 = (19)(13\(140)(n) n=231 say 3 Use n= 5 rivets for member BC, ® Average tensile stress for member BC. ait = An = 864 - 1946) Ay = 780mm? ae For tension members we use the net area. g = 9500 750 $= 124 MPa @ Average compressive stress for member BE. FEZ For compressive member, use gross area, P San cae 1780 S= 449. MPa ‘A truss joint shown consists of a bottom chord C made up of two angles, and web members A and B each carrying the given loads. Using A 502-1 rivets with an allowable shearing and bearing stresses of 120 MPa and 600 MPa respectively, and AISC specifications. @ Which of the following gives the requited mamber of 18 mm diameter rivets to develop fully the truss joint for member A. @ Which of the following gives the requited number of 18mm diameter rivets for member B. ® Which of the following gives the required number of 18 mm diameter rivets for the bottom chord C. Solution: ® Forsection A: Shear of rivets (double shear) TRAgSs 150000 = (n/4) (18) (n)(2(120) n=246 say 3 rivets Bearing of rivets on the gusset plate: T= ApSp 150000 = 18(9.5)(600) in n=146say2 Use 3 rivets for section A. ® For section B Shear of rivets: (double shear) Ts AeSe 220000 = (7/4) (18)? (2)(n)(120) n=36sayd Bearing of rivets on the pusset plate: T=ALSp 220000 = 18(9.5)(n)(600) ne2l4 say 3 ‘Use 4 rivets for section B. ® For section (Bottom chord) ‘Shear of rivets (double shear) Te Ass. T = 624.7 - 400 TH 224.7 KN 224700 = (m/4) (18)? (2)(n)(120) n=3.68 say 4 Bearing of rivets on the gusset plate: T=A,S 224700 = 18(9.5)(n)(600) n=219 say For symmetry reasons use 5 rivets placing one rivet at the intersection of the rivet gage of sections A and B. aoa ed RAL ‘Show that the stress in a thin-walled spherical shell of diameter and wall thickness t subjected to internal pressure Pis given by S= 4. EAL A cylindrical pressure vessel is fabricated from steel plates which have a thickness of 20mm. The diameter of the pressure vessel is 500 mm and its: length is 3.m. Determine the maximum internal pressure which can be applied if the stress in the steel is limited to 140 MPa. Solution: F=pDL P-(500)(3000) F = 1,500,000 p'N 2T =F = 1,500,000 p T = 750,000 p pa: oa 7501 MO = 720)(3000) p=11.2 MPa [simi s7aEes) EAL © Find the limiting peripheral velocity of a rotating steel ring if the allowable stress is 140 MN/m# and the mass density of steel is 7850 kg/m. @ At what angular velocity will the stress reach 200 MN/m? if the mean radius is 250 mm? ® If the angular velocity is 740 rad/sec. and the mean radius is 300 mm, what is the peripheral velocity? Solution: ® Limiting peripheral velocity. o= pvt 140000000 = 7850v? Ws 133.5 m/s = pvt 200000000 = 7B50v? v= 15962 m/s vy 159.62 onT 028 = 635.5 radisec, ® Peripheral velocity. oat Te ¥ FO = 35 Va 222 mis Pe A water tankis 8 m. in diameter and 12m. high. the tank is tobe completely filled. © Determine the tangential force on the tank. ®@ Determine the minimum thickness of the tank plating if the stress is limited to 40 MPa. Solution: © Tangential force on the tank. Fewha F = 9910{12)(8)(0.001) P= 941.76 N 2P =F = 41.76 P= 470.88 N @ Minimum thickness of the tank. s-2 x 470.88 = OD) ta 177 mm TEI ‘The strength per meter of the longitudinal joint in the figure is 480 KN, whereas for the girth joint it is 200 KN. Determine the maximum diameter of the cylindrical tank if the internal pressure is 15 MN/m2, Solution: For longitudinal joint Fu2T pDL=2T (1.5 x 10° (DML) = 2(480)(1000) D=0.64 m. T = (x Dy(200)(1000) N FeT p 2 = x D)200,000) (15x 10% P= 200,000 D=053m. UseD=0.53 mi, (maximum diameter) 50 | Problem A pipe carrying steam at 3.5 MPa has an outside diameter of 450 mm and a wall thickness of 10:mm. A gasket is inserted between the flange at one end of the pipe and a flat plate used to cap the end. @ How many 40 mm diameter bolts must bé used to hold the cap on if the allowable stress in the bolts is 80 MPa of which 55 MPa is the initial stress? ® What circumferential stress in developed in the pipe? @. Why is it necessary to tighten the bolts initially, and what will happen if the steam pressure should cause the stress in the bolts to be twice the value of the initial stress? Solttion: ® F=5)G) (4307 F = 503,270 N Bolt stress due to steam pressure § = Final stress - Initial stress S$ =80-55 $=25MPa T=AS # =] (0)? @5) F=31416N n=162 Use n= 17 bolts 5 = 75.25 MPa @ The bolts should be tightened initially to seal the gasket in order to control the steam _ pressure and when the steam pressure cause the stress in the bolts to be doubled, the bolts will tend to elongate thereby causing 2 gap on the gasket which is no longer effective in sealing the steam pressure. A spiral-riveted penstock 1.5 m. in diameter is made of steel plate 10 mm thick. The pitch of the spiral or helix is 3m. The spiral seam is a single-riveted lap joint consisting of 20 mm diameter rivets. Using 5, = 70 MPa and S, = MPa. ® Compute the strength of the longitudinal joint, ® Determine the spacing of the rivets along the seam for a water pressure of 1.25 MPa. Neglect end thrust. @ What is the circumferential stress? Solution: © Strength of the longitudinal joint. Shearing of rivets: T=A,5, =] Go? 7) T=21,991N Bearing of rivets T=A,S T = (10)(20)(140) T = 28,000N Use T = 21,991 N ® Spacing of the rivets along the seam. a = VGSny + (3)? a = 5.586 m. QP FePDL 2(21,991) = (1.25)(1500)(L) L = 2846 mm 5.586 ce bes. 5.586(23.46) $= 3 S=43.7 mm @ Circumferential stress, ToAS, 21,991 = (10}(23.46) & S,= 93.74 MPa 30 mm diameter rivets, ‘Neglect end thrust. Solution: Shearing of rivets: TH AsSs TF 30} (70) T = 49,480 N Bearing of rivets: T= ApSy T = (30)(10)(140) T=42,000N Use T = 42,000 N ® Spacing of the rivets. a=V(n)? + 3) = 6.963 m, 27 =F =PDL 2(42,000) = (1-25)(2000) L L=36mm § = {6:963)(336 3 $=7799 mm UseS = 78 mm ® Circumferential stress. TaASs 42,000 = (10)(33.6) 5, Sy= 125 MPa @ Compute the strength of the longitudinal joint @ Determine the spacing of the rivets along the seam for a water pressure of 1.25 MPa. @ What is the circumferential stress? © Strength of the longitudinal joint. A spiral-riveted penstock 1,5 m. in diameter is made of steel plate 10 mm thick. The pitch of the spiral or helix is 3 m. The spiral seam Is a single-riveted lap joint consisting of 20:mm diameter rivets, Using a 2 m, diameter penstock fastened with = ‘The tank shown in the figure is fabricated from 10 mm steel plate, Determine the maximum longitudinal and circumferential stresses caused by an intemal pressure of 1.2 MPa. © Compute the longitudinal strength of the tank. @ Determine the max. longitudinal stress caused by an internal pressure of 1.2 MPa. ® Determine the max: creumferential stress caused by an internal pressure of 1.2 MPa Solution: © Longitudinal strength. T F=pa F = (1.2) [:400) (600) + 2 oo] F = 438,796N ® Longitudinal stress. FuAS, 438,796 = [(600)(2)(10) + (400R)(10)] 5. Se= 17.86 MPa @ Circumferential stress. F = (1.2)(2000L) F=1200LN 2T=F=1200L T=G00LN T=AS, 600L = (10L)(S) 5,2 60 MPa re ec) The tank shown in the figure is fabricated from steel plate. Determine the minimum thickness of plate which may be used if the stress is limited to 40 MN,/m? and the internal pressure is 1.5 MN /m?. Solution: Longitudinal stress; F = 1,5(400(600) + (40032) F= SHM96.N 548496 = [600(2)t + 400n(t)] (40) t=558 mm. Use t = 18.75 min. During a stress-strain test, the unit of deformation at a stress of 35 MN/m? was ‘observed to be 167 x 10 m/'m and ata stress of 140 MN/m? itwas 667 x 10° m/m. If the proportional limit was 200 MN/m?, @ What is the modulus of elasticity? @ What is the strain corresponding to a stress af 8) MN /m?? Would these results be valid if the proportional limit were 150 MN/m?? Solution: @ Modulus of elasticity. AE = (667 - 167) x 10% A€=500x10%m/m AS= (140-35) AS= 105 MN/m* a pu 105s 108 500 x 10% E= 210% 10? N/m* E=210GPa @ Strain corresponding to a stress of 80 MN/m?. SEE) 80 x 10 = (210 x 10%) E 6 =381x10%m/m @ Validity of results if proportional limit is 150 MN /‘m?, The results are still valid because the stresses given are below this limit. ao 1000)(150)(1000) (300)(200 x 10°) A uniform bar of length L, cross-sectional area A, and a unit mass p is suspended vertically from one end. Show that its total elongation is 5 = pg L?/2E. if the total §450mm mass of the bar is M, show also that 8 = Mg L/2AE. 2 : §=4.33 +50 Solution: = sc Pk = 58.33 ie AE Videos “ , : . i = PB Asd, d= AE L I A steel wire 10 m. long hanging vertically supports a tensile load of 2000 N. pg Neglecting the weight of the wire, determine the required diameter if the stress is not to aE fe ' exceed 140 MPa andthe total elongation is nat exceed Samm, Assume E = 200 GPa. o Solution: a=ff b s]; From stress: Peas a8 Ae noo = (Fd) (140) M=pAL d=426mm 2 PAL From elongation, —MgL -fk o=TAE “AE __ (2000)(10)(1000) % (Fa?) (200 x 10%) an d=505mm A steel rod having a cross-sectional area of 300 mm? and a length of 150 m. is Use d= 5.05 ns suspended vertically from one end. It supports aload of 20 kN at the lower end. If the unit mass of steel is 7850 kg/m? and E = 200 x 10° MN/m?, find the total elongation of the rod, a Ee Solution: Assteel tire, 10 mm thick, 8. mm wide, and of 1509 mm inside diameter, is heated and §=8, +5; shrunk unto a steel whee! 1500.5 mm in diameter. If the coefficient of static friction is pel? ee 0.30, what torque is required to twist the tite relative to the wheel. Use E = 200 GPa. , & oe 2E : _ (7850)(9.81150) ae ‘2 (200 x ine boa 3 =433mm (1500.5 - 1500) kN | 5 1571 mm L=15002 mm A= (60)(10) mm? _—_(Mas00n) 1971 = [00)(200 x 105) T= 53,333N 2T=pDL 2(53.333) = p(1500)(80) p= 0.889 MPa NN = (0.889)(m}(1500)(80) N= 335,101 N FeuN F = 0.30(335,101) F = 100,530 N = 100.5 KN. Torque = F(0.750) Torque = (100.5)(0.750) Torque = 75.4 kNim Ea feed itast) An aluminum bar having a cross-sectional area of 160 mm? carries the axial loads at the positions shown in the figure. If E = 70 EPa, compute the total deformation of the bar, Assume that the baris suitably braced to prevent buckling, Solution: gutk “AE 35k I5kN sou ok = (3500091800) = * asoy(70x103) 9" boam——}-— om —L ton} = §20,000)(1000) _ #2" Tea\79 x 108) 1 _ fto,00n7(600) _ ®3= F160)(70 x 108) 0 Total deformation: 5 =6)+5)+53 8225+ 1786-0536 6 = 3.75 mm (elongation) Ifthe magnitudes of the loads at the ends are interchanged, ie. if the load at the left end is 10 KN and that at the right end is 35 kN, compute the total deformation of the bar. Assume that the bar is suitably braced to prevent buckling, Solution: PL S=aE '10,000)(800) Se naaeath wists sou any &=0714 mm gene Poe tte pen] = Pee TORN tee (160)(70 x 107) Esau) §)= 0.446 mm _ {35,000)(600) 3 (160)(70 x 103) = 1.875 mm Total deformation, B= 8-52-55 6 = 0,714 - 0.446 - 1.875 6=1.61 mm (contraction) An aluminum tube is fastened between a steel rod and a bronze rod as shown, Axial loads are applied at the positions indicated. Assume that the assembly is suitably braced to prevent buckling and that E, = 200x 10? MN/m?, E, = 70x 10? MN/m?, and E, = 83x 10? MN/mé_ ® Find the value of the force acting on the bronze so that will not exceed a maximum overall deformation of 2 mm or a stress in the bronze of 120 MN/'m?. ® Find the value of the force acting on the steel so that will not exceed a maximum ‘overall deformation of 2 mm or a stress in the steel of 140 MN/m?. Find the value of the force acting on the aluminum so that will not excece a maximum overall deformation of 2 mm ora stress in the aluminum of 80 MN /m?. 60 Solution: From total deformation 5=6, 8 =8,- 6-8, (2P)(800) (300)(200 x 10") 6, = (2.67 x 105) P ae ~ (450)(83 % 107) 8p = (4.82 x 10°) P = 6, = (4.76 x 10°) P 8 = (2.67 x 105) P - (4.82 x 104) P - (4.76 10) P =~ (691 x 105) P (contraction) Allowable 8 = 2mm. 2= (6.91 105) P P=28,900N From strength of each member: PeAS For Bronze Py= ApS 3P = (450)(120) P= 18,000N For steel: By= Ass; 2P = (300)(140), P= 21,000 ‘Therefore, safe axial load P= 18,000 N Force acting on the bronze. Fe3P. F = 3(18000) F = 54000 N ® Force acting on the steel. F=aP F = 2(18000) F =36000.N @ Force acting on aluminum: F=2P F = 218000) F = 36000 N Cele ‘The rigid bars shown in the figure are separated by a roller at C.and pinned at A and D. Assteel rod at B helps support the load of 50 KN. Compute the vertical displacement of the roller at'C. Solution: IM, =0 P(3) = 25(4.5) PSa75kN PL b=nE 5, = -(27500)(3000 * (300)(200 x 107) 6) = 1.875 mm, By ratio and proportion: ee &, - ge) (45) 8, = 2.8125 mm Avuniform concrete slab of mass M is to be attached as shown in the figure, to two rods whose lawer ends are initially at the same level. Determine the ratio of the areas af the rods so that the slab will remain level after it is attached to the rods. Solution: =F =M,=0 P, (5) = w (3) 67 (SaerESTa) ‘The rigid bar AB, attached to two vertical rods as shawn in the figure, is horizontal before the load is applied. If the load P = 50 KN. ® Determine the force carried by the aluminum. ® Determine the force carried by the steel @ Compute the vertical movement under the load. Solution: @ Force cared by the aluminum. Ma =0 P, (5) = 50(2) P,=20kN PaSDKN ® Force carried by the steel. Py +P," 50 P,=50-20 Py=30 kN @ Vertical movement under the load. = {30,000)(3000) ®5= (300)(200 x 10°) §=15mm ~ 120.000)(4000 8 (5OO)(70 x 104) 8, = 2.266 mm 5, -8, = 0,766 mm 0.766 _ 6-5, cg 3y- 8,= 2 (0.786) 8.2031 +1.5 ape 1.814 ma | Lae | ene The rigid bars AB and CD shown in the figure are supported by pins at A and Cand ine the maximum force P which can be applied as shown if its the two rods, Det vertical movement is limited to 5 mun: Negleet the weights af all members. Solution: ‘rin EM,=0 s Alor | P, (6) =P) oat Pp Ras EM,=0 P, (3) =P, (6) n @@ PsP ott ORE (2000) (500)(70 x 10%) 6, = (5.714 x 10°) P mm (F) e000) (G00}(200 x 105) 8,= (1667 105) Pmm A round bar of length L tapers uniforinly from a diameter D at one end to a smaller diameter d at the other. Determine the elongation caused by an axial tensileload P. Solution: PL 7" AE dee Raayyre paeyre L J [at. + (o-d)x]? axe = Soh “5 [(au-+ (DO - dye" ~d) a flare: Dalpe feos lacrb-ac- a st acl A uniform slender rod of length L and cross-sectional area A is rotating in a horizontal plane about a vertical axis through one end. If the unit mass of the rod is P, and it is rotating at a constant angular velocity of w rad/sec, show that the total elongation of the rod is p w* L3/3E Solution: Pema, P= (p Ax){(rw2) P= (Ax) (L-3) a? ee pe PAS lx -x] t oe pe we = [ux x ea as SaiEe As shown in the figure, two aluminum rods AB and BC, hinged to rigid supports, are pinned together at B to carry a vertical load P= 20 KN. If each rod has a cross- sectional area of 400: mm? and E = 70x 107 MN/m?. Assume o = 30° and 0 = 30". Compute the deformation of rod AB. ‘Compute the deformation of rod BC Compute the horizontal displacement afjoint B B ‘Compute the vertical displacement of joint B. uu P=20kN a7 Solution: © Deformation of rad AB: “(40070 x 10 S4u= 2.143 mm (elongation) ha N ® Deformation of rod BC. Pataki Sige wf28200002000) Be “(A00}(70 x 10°) Sue Sye = 1.429 mm (contraction) Pet) @ Horizontal displacement of B. ! 2443. Cos (120-8) = 1429. Cos (120-0) 2.143 1.429 Cose Cos (120-0) 1.5 Cos (120-9) = Cosa 1.5 [Cos 120 Cos » + Sin 120 Sino] = Cuse 151-05 Cos a + 0.866 Sine] = Cosa = 0.75 Cos 0 +1.299Sina =Cos@ 1.200Sin © =1.75Cose sp l823 bios tne = = 53.41" oe & = SBSin & $y = 3.595 Sin 6.587" G, = 0.412 mm (leftward) @ Vertical displacement of joint B | 8, = SBCos 8 | §, = 8.595 Cos 6587 §,=3.57D mm (downward) As shown in the figure, rod AB is steel and rod BC is an aluminum, are hinged to rigid Supports which are pinned together at B to carry a vertical load P = 20 KN. Each rod has a €ross sectional area of 400 mm? and E, = 200000 MN/m? and 70000 MNV/m#. Assume a = 45° and 8 = ® Compute the deformation of steel ® Compute the deformation of aluminum. ® Compute the vertical displacement of point B. @ Compute the horizontal displacement of point B. Solution: ® Deformation of steel, Eyed AB Sin 75° = 20 Sin 60° AB = 17.932 kN Ty0 E 6 30" = 17.932 Cos 45° 4.641 KN AE ._ (17,932)@000) gee (400)(2000 * 10%} SAB = 0.672 mmr (elongation) oo @ Deformation of aluminum _ftd6419(2000), ~~ (400)(70 x 10°) SBC = 1.046 mm (contraction) ® Vertical displacement of B. Cos 0 2 Cos (105-0) =p Cosa _ 0.672 Cos (105-6) 1.046 1.557 Cos a = Cas (105-a) 1,557 Cos a = Cos 105 Cos a + Sin 105 Sino 1,557 Cos @ = - 0.259 Cos o + 0.966 Sin o 1.816 Cos @ = 0.966 Sine tan.e = 0.966 Sine L816 tan © =a 566 @=62" +4520 G262-45=177 0.672 0.672 SB=Cose “Cos 62 3y = SB Cos 8 6) = 1.431-Cos 17" = 1.369 mm (downward) @ Horizontal displacement of B. 3, =SBSine y= 1431 Sin 17" y= 0.418 mm (righward) elongation of a cone suspended from its base. Solution: Fora tee terete cone veh int 2+ Rr) 2 VER O90) *-oxa | [(D +x + ded +x +a] Weng SF [(d+xP+da+n+e?] afar id + xJP+ did +x) 4d Per Cat] asta Day w = PRY (ari : cs [dL * (d-ay yy = PRY jOUEe za (D- diy + (D- dj? B= az SE 1 pEy {Os 2+ 3dL (D- dy + 3d [dL + (B-d)y] a fscro=c) ee [ats ©-aii) L W Bey (ee dly[+ dt. ae dy +l) + (db? + db (D- diy + (dl? [dL + (D = djy} } A round bar of length L, tapering uniformly from a diameter Dat ance end to a smaller diameter d at the other, is suspended vertically from the large end. If P is the unit mass, find the elongation caused by its own weight, Use this result to determine the } D-d) SdL(D- dy? y? + 3(d LF? (D- di) “303 | : [al +(D- ay } a A W_ ps ee ESTES A Ww. A “HO-d) UYo-djy +db) spa — fan “0. j {ke- eyralle [D- earl saa dL)? “sD a xtra {te yt To. ay al } py i=5p ae s [(D-a)y+at]- cE - diy edytalft at ~3D- Pg (U 2 i. *O-dy+ rath aj + AL) aL (dL) (aL ke ‘ape (urate *D-di+at {eb 6") DLE via dL 2} =o dy Fal z ee +(auF} 212 Bi is eee = [ose | MeL] Pr? 4 2d912 — spd? 2 eS 70-8 ; Ee i 2d ] gaPel D+ 2d? - al = 2 DyD- dj? pel* p(D-dyb + bd - 2d?) or [ Dib: dF ] gale? Dee Dd 2d) = 56 Bib=a) pet DipD+d) 2a] 55 [ DD | ope Bo+d eet f Peat se GE [en | eee belt Ded) at on FED) 3ERD-S Fora cone, d=0 (+0) pg lear 5-Pheo-o SED =o) =a Ge GE A solid cylinder of diameter d carries an axial diameter is 4 Pv/n Ed, Solution: (Ascumea tensile load) eo P bs er diameter. Ss 4p nee A rectangular aluminum block is 100 mm long in the X direction, 75 mm wide in the Y direction and 50 mm thick in the Z direction, It is subjected to a tri-axial loading consisting; of a uniformly distributed tensile force of 200 kN in the X direction and uniformly distributed compressive forces of 160 KN in the ¥ direction and 220 KN in the 2 direction. [fv = 1/3and E = 70 GPa, determine a single distributed loading in the % direction that would produce the same Z deformation as the original loading. load P. Show that its. change in Solution: For tri-axial tensile stresses. E,=p [5-1 6+ 5)] g, = 200(1000) = =S0}(75} $= 5333 MPa (4) _ 1 60(1000) Sy =7100)(60) $)=32MPa 22011000) S:= T0075) $,=2933MPa () 1 1 - eae agshge [6292-3 63-23-32] - 5.206 x 10% €, = 15618 x 10-9 S,=6,E 5, = 1.5618(10)"2 (70000) 5, = 109.326 MPs Py™ Axo P, = 75(50)(109.326) P, = 409973 N Py = 410 kN EAs, A welded steel cylindrical drum made of 19-mm plate has an internal diameter af 1.20'm. By how much will the diameter be changfed by an internal pressure of 1.5 MPa? ‘Assume that Poisson's ratio is 0.30 and E = 200 GPa, Solution: gable. Considering end thrust; Cae ee aa ATE OYE - 90 0.3(45) = 90 MPa "= 700000 ° 200000 = 3825 x 106+ aD=De, AD = 1200(3.825)10°¢ AD = 0.459 nam 9, yee o a fap ve aD aoe aD= De Neglecting end thrust; a0 &=2 -0 90 = 200000 AD=Dy, 1200/90) 4D = 500000 4D=-05mm 75 [SUMPLE STRAT) Ea A 50-mm-diameter steal tube with a wall thickness of 2 mm just fits ina rigid hole. Find the tangential stress if an axial compressive load of 10 kN is applied. Assume v =0.30 and E =200 x 107 N//m?, Neglect the possibility of buckling, Solution: Because of restraint exerted! by wall of rigid hole ¢, = 0 5-7 l-vaL + 9] g, = 0 (contact pressure at outside but zero at inside of tube) O=bla-wWeL+O) -vaped P Sq ze ony 10000 Le" n(50)(2) 4 oy = 31.83 MPa Oevay = 0.3(31.83) 0, =955.MPa (tangential stress) A 150-mm long bronze tube, closed at its ends, is 80 mm is diameter and has a wall thickness of 3 mum. If fits without clearance in an 80 mm hole ina rigid block. The tube is then subjected to an internal pressure of 4.00 MN/m*. Assuming ¥ =z and E= 63x 10° MN/m?, determine the tangential stress in the tube. Solution: fy Ey -v0,) y= 2 (a-vo) € =0 (because of constraint of rigid block) a=Vvo, ane (80) OL = a3} oy = 26.67 MPa Gave, a. =4 (2667) 9, =8.89MPa (tangential stress in the tube) EAL A ssteel bar 50mm in diameter and 2 m. long is surrounded by a shell of cast iron 5mm thick. Compute the load that will compress the combined bar a total of 1 mm in the length of 2m. For steel, E = 20x 10” N/m2, and for cast iron, E = 100x 10? N/m* Solution: PL be aE A, =5 (50)? ee irom A, 2625 tmm A=} (OF - 625 x Ag=22mm* 8,=8g=1mm p=——_fe(2000) © (625n}(200 x 105) P, = 196,350 N ‘ Pg (2000) © (Q75x\(100 x 103) Py = 45,197N PalePy P = 196,350 + 43,197 P=B957N P= 240k A reinforced concrete column 250 mm in diameter is designed to carry on axial compressive load of 400 KN. Using allowable stresses of S,= 6 MPa and 5; = 120 MPa, determine the required area of reinforcing steel. Assume that E,= 14 GPa and E,= 200 GPa, 5, = (14.29)(6) S,= 65.71 MPa < 120 MPa P, + Pp= 400,000 9A, +5. Ac = 400,000 Acs (507 A, “Ac= 15625 0- Ay 85.71 A, + 6(1525 - A,) = 400,000 7971 A, = 400,000 - (15625 2) Ag= 1323 min? A timber block 250 mm square is supported on each side by a steel plate 250 mm wide and {mm thick, Determine the thickness t so that the assembly will support an axial load of 1200 KN without exceeding a maximum timber stress of 8 MN/m? ot a maximum. Steel stress of 140 MN/m®” For timber, E = 10 x 10? MN/m?, for steel, E=200x10°MN/m? [Simerte STAT) Solution: 5, = 160 MPa > 140 MPa (Stee! fails) Use S,= 140 MPa 140 -= 20 Sy, S-=7 MPa <8 MPa (safe) Pet Py = 12001000) AgSs + Aye Spe = 1,200,000 4(250t)(140) + (250)(250)(7) = 1,200,000 t= 5.45 non A rigid block of mass M is supported by three symmetrically spaced rods as shewn in the figure, Each copper rod has an area of 900 mm?; E = 120 GPa: and the allowable stress is 70 MPa. The steel rod has an area of 1200 mm#; E = 200 GPa; and the allowable stress is 140 MPa. Determine the largest mass M which can be supported. Solution: 5,= 1.1115, when $.= 70 MPa S,= 77.778 MPa < 140 MPa WP, +2P, WeAS. +2 A,5, Mg= (1200)(77.778) + 2(900(70) ‘M(9.81) = 219,333 N M = 22,358 kg = 22.3 x 10) kg A rigid block of mass M is supported by three symmetrically spaced rods as shown in the figure. Each copper rod has an area of 900 mm’; E = 120 GPa; and the allowable Stress is 70 MPa. The steel rod has an area of 1200 mm: E = 200 GPa: and the allowable stress is 140 MPa. How should the length of the steel rod be changed so that each material will be stressed to its allowable limit? 140 L, _ 70161 200 bo L= 133.33 mm RAZ: The lower ends of the three bars in the figure are at the same level before the rigid homogenous 18 Mg block is attached. Each steel bar has an area of 600 rum and E = 200 GN/m?., For the bronze bay, the area is 900 mm? and E = 83 GN/m, Find the stresses developed in each bar. Solution: S5= 8, Sp Te Ey Ep S21) _ Sp (1.6) 200 &. $,= 3.855 5, Pp+2Py2W Ay Sy + 2A, 5, = (18,000)(9) 900 S, + 24600)(3.855 S,) = (18,000)(9.81) S,= 32 MPa S,= 3.855 (32) 5, = 123.4 MPa ‘The rigid platform in the figure has negligible mass and rests on two aluminum bars, each 250 mim long. The center bar is stee! and is 249.90 mm long, Find the stress in the steel bar after the center load P = 400 kN is applied. Each aluminum bar has an area of 1200 mm? and E = 70 GPa. The steel bar has an area of 2400 mm? and E = 200 GPa. Solution: P=t0OkN Su soe 8,284 010 SL _SL oan 0.10 5,050) _ 5,490 7x10? 200% 198 * 920 5, = 0.34986 S, 4-28 Alumina. ‘Alurvinwart P,+2P, = 400(1000) AgSs +2 A, S, = 400,000 2400 5, + 2(1200)(0.34986 5, + 28) = 400,000 5, + 0.24986 S,+ 28 = 166.67 $,= 102.73 MPa HZ» Three steel eye-bars, each 100 mm by 25 mmvin section, are to beassembled by drivi 20 mm diameter drift pins through holes drilled in the ends of the bars. The center-line spacing between the holes is 10m. in the outer two bars but is 1.25 mm shorter in the middle bar. Find the shearing stress developed in the drift pins. Neglect local deformation at the holes and use E, = 200 GPa. =o Tat Po Pm Py Solution: By + By = 125mm a re =135 Py (10,000)__ | __ Po (10,000) _ (2500)(200 x 107) ~ (25000)(200 x 1 Ppa + Po = 62,500 Po Pu Pe smidile bar is 1.23 sem shorter Bet hades As shown in the figure, three steel wires, each 30 mm? in area are used to lifta mass M Their unstretched lengths are 19-994 m., 19.997 m., and 20,000 m. @ IfM = 600 kg, what stress exists in the longest wire? ® IfM=200kg, determine the stress in the shortest wire. Use E = 200 GN/m?, Solution: © Stress exists in the longest wire Determine first the force P, and P; to bring all wires to a length of 20,000 mm. 8, = 20000- 19994 = 6 mm PyL ele Kye, _-P. (9.994) ~ (30)(200 x 103) Py = 18005 N 3) = 20000 - 19997 = 3mm Pol = AOE, __Py (19,997) >= Go}(200 x 105) Py = 900.1.N When the lengths of the wires are the same, each will carry equal loads. py2 bP Py = O00H3.81) - 1800.5 - 900.1 a = P,= 1061.8N ® Sress in the shortest wire. IfM 2200 kj Pye (9. = 100) -90n P3=-738.6N (remains slack} p, —200(9.81) - 900.1 -_ 2 P)=530.95N P, = 530.95 + 900.1 Py = 1431,05N 5, = 2408 $1= 47.70 MPa The Assembly in the figure consists of a rigid bar AB (having negligible mass pinned at Qand attached to the aluminum rod and the steel rod. In the position shown, the bar AB is horizontal and there is a gap d = 4 mm between the lower end of the aluminum tod and its pin support at D. Find the stress in the steel rod when the lower end of the aluminum rod is pinned to the support at D. Solution: S455, 06712 25, 24-5, 25, +S,=4 PL) , (PL 2(x#),+ Ge), a4 2 P1500) PA500) _ 300(200000) * 400(70000) = 5x 105 P,+536x105P,=4 EM, =0 P,{0.6) = P,(1.2) P,=2P, 5x 105 QP) 4536x105 P,=4 P, = 26046.5 N P, = 2(26046.5) FOr ay P, = 52093 N ' Sea _ 52093 Sy="300 P, Ps $, = 173.6 MPa 5, = 174 MPa A homogeneous rod of constant cross-section is attached to unyielding supports. it catries an axial load P applied as shown in the figure. Prove that the reactions ace given by Ry = Pb/L and Ry = Pa/L. Solution: 5, = 83 Hele) ES RE: AE AE Rya=Pb-Rib Ry (at b)e Pb 5 citi? A homogeneous bar with a cross-sectional area of 500 mm? is attached to rigid suports. It carries the axial loads P, = 25 kN and P, = 50 kN, applied as shown. Determine the stress in the segment BC. (Hint: Use the results of previous problem, and campute the reactions caused by P; and P, acting separately. Then use the principle of super- Position to compute the reactions when both loads are applied). Solution: 7 Ry=Ry' +R2 i i nyo Ppb Baba " ; ey Fie itis Tie ee 25)(2.10) _ (50)(0:90: y 3 Ry = 36.11 kN Force acting an BC = 36.11 - 25 = 11,1 kN P s== Ti gu HLALaOD) sew ELAN $= 22.22 MPa [eee A B Rae ‘The bar shown is firmly attached to unyielding supports. Wa load P = 200 KN is applied. D Find the load carried by the aluminum ® Find the load carried by the steel. ® Find the stress in aluminum. ® Find the stress in steel. Solution: ® Load carried by aluminum. = 52 +R (200) _ (200- Ry)(300) 6 (900)(70) ~~ (1200)(200) 2.54 Ri = 200-R, ; 7 Ry =565 kN ® Loadcarried by steel. Ry = 200-R, Ry = 200- 56.5 Rp= 143.5 KN @ Stress in aluminum. 5, = 25000) - 62.8 MPa ®@ Stress in steel. $,-HS3H000) | 719.6 MPa What maximum load P can be applied without exceeding an allowable stress of 70 MPa for aluminum or 120 MPa for steel? Can a larger load P be carried if the length of the aluminum rod be changed, the length of the steel portion being kept the same? Ifso, determine this length Solution: Since 5, = 1196 MPa = 120 MPa, therefore maximum load P = 200 KN A larger load P can be carried if the aluminum and steel portions will each the maximum allowable stresses simultaneously ene, Saba Syl BE, Es 70(LA) _ 120(300) 0 ~~ 200 LA = 130 mn A rod is composed of three segments shown in the figure and earries the axial loads P= 120 KN and P) = 50.KN. 1F the walls are rigid. @D Determine the stress of bronze. ® Determine the stress of aluminum, © Determine the stress of steel. ah 6rs B10Gh E2000 Solution: AS200 min? ASI20 mim? Ano mat @ Stress of bronze. Ramee, Ry (600) (120-Ry)400) | (170- R,) (300) (2400)(83) = (200070) — + Teagan) Ry=97kN Ciies— Ry =170-R, Ro = 170-97 Rom 73 kN For Bronze: 170-R= 1. San SUM gos MPa 4 Z @ Stress ofaluminum. Foraluminum: P= 120-Ry'= 120-97 P=BkN Sa = 23000) . 9.07 MPa @ Stress of steel, For steel: 3(1000. S.= “hog = 121.67 MPa A rod is composed of three segments shawn and carties the axial loads Py = 120 KN and P, = 50 KN, If the left wall yields 0.60 mm. @® Determine the stress of bronze, @ Determine the stress of steel. Determine the stress of aluminum. Fired rome Bat). GPx Bet0GPs | E=200 GPs Solution: AS2400 ment Aw 1200? A=f0 ent @ Stress of bronze. 5, + 0.60 = 82+ 55 R; (600) (120 - RyW400) (170 - RyH300) Ganoyeny © Trson roy ceDo}e200) Ry = 38.586 kN Ry= 170 Ry Ro = 170 - 36.586 Ra= 131.414 KN _38.586(1000) _ Ss=" “5409 = 1608 MPa ® Stress of steel. _ 131.414(1000) | Sy B00 219.02 MPa ® Stress of aluminum, For alurninam: P= 120-Ry P= 120 - 38.586 P=8Lald kN 5,» #1.414(1000) x 1200 5, = 67.85 MPa 89 Siti? Enz A steel tube 2.5 mm thick just fits over an aluminum tube 25 mm thick. Ifthe contact diameter is 100 mm. and the outward radial pressure on the aluminum tube is p=4MN/m?. E, = 200000 MPa, E, = 70000 MPa @ Compute the contact pressure. @ Compute the tangential stress on the steel tube. @ Compute the tangential stress in the aluminum tube. Solution: ae ey @ Contact pressure P. For steel tube: 2P,=F; alain 2P, = P{DML) (2 Sem hick) OL Pah & For aluminurn tube: sre! 2P, = Fy 2P, = (p- PDL Fate pw PDL ci 3 5 Fy Fee Be P, P.=Hp Pe 2D. 2.96 MPa (contact pressure) @ Tangential s PD Sor Gish 364 a = 59.2 MPa @ Tangential stress of aluminum tube. (p-P)D Sa= it p FBO _ 208 MPa A.steel tube has a thickness of 251mm and that of aluminum tube is also 2.5 mm. thick. ‘There is a radial clearance of 0.01 mm. between the tubes before the internal pressure of 4 MN/m? is applied to the aluminum’ tube. The contact diameter is 100 mm and E, = 200000 MPa, F, = 70000 MPa. ® Compute the contact pressure. ® Compute the tangential stress of steel, @ Compute the tangential stress of aluminum. Solution: D Contact pressure. Compute the pressure to make contact fora clearance of 001 mm, i zr 3(100) 230.01 = 300) S=14MPa a P. = 7000 ~ F000 2018.3 -P)=7P, 27 P. = 203.3) P. = 244 MPa (contact pressure) 33-P) @® Tangential stress of ‘steel. 2bD erly 2.440100) _ Spe 75) = 18-8 MPa @ Tangential stress of aluminum. _(4-2.44)(100) S,= Fat 512M Pes P, Py Pes oT [Siar Tait In the assembly of the bronze tube and steel bolt shown, the pitch of the bolt thread is 0.80 mm and the cross-sectional area of the bronze tube is 900 mm? and of the steel bolt §9450 mm?, The nut is turned until there is a compressive stress of 30 MN/m? in the bronze tube. Find the stress in the bronze tube if the nut is then given ane additional turn. How many turns of the nut will reduce this stress to zero? E, = 83 GPa, E,= 200 GPa, Solution: For one additional turn of the nut 0.80 = 8, +5, wm (7),-(@), 080 = aT Te tea 800 =45, +9645, r=, ApSy= A, 5, 900 S = 4505, S5= 25, B00 = 4(2 Sy} + 9. Sy ‘Sp = 45.35 MPa Total stress ‘Sp = 30+ 45.35 = 75.35 MPa To. reduce S, to zero, required number of turns: nn POSS = 1.66 turns [ Simivie sani) As shown in the figure, a rigid beam with negligible mass is pinned at 0 and supported by two rods, identical except for length. IP = 30 kN. @ Find the load in rad A. @ Find the load in rod B. @ If rod A elongates by 2 mm., how much is the elongation of rod B. Solution: @ Load inrod A. p= L755, Pen Sie ) 1.75 Py (15) fs A Pg=13125P, =EM,=0 Pa 2) +Pp 35) = 3002) 2P, +3.5(1.3125 Py) 60 Pq, = 910 KN ® Load inrod B. Pa = 1.3125 (9:10) Pp= 198k ® Elongation in rod B. Sp = L758, Sy = 1.752) y= 3.5 mm As shown in the figure, a rigid beam with negligible mass is pinned atone end and supported by two rods. The beam was initially horizontal before the load was applied. If P= 120 kN, @ Find the elongation of steel. @ Find the elongation of aluminum. @ Find the vertical movement af P. Solution: © Elongation of steel 5.5 63. 8, = 28, P,(3) 2, (4) 500(70) ~ 600(200) P,=14P, EM, =0 P,Q) + Pa(6)= PCS) 3P, + 6¢1.4P,) = 120(5) P,=5263KN 52.68 (4000 5 = EparatO) = 1.75 reat @ Elongation of aluminum, 5, = 28, 4, = 201.75) = 3.5 mm © Vertical movernent of P. uae $73 8, = 217) 2.92 mm A rigid bar of negligible mass, pinned at one end, is supported by a steel rod and a bronze rod as shown. ® Compute the load carried by the steel without exceeding a stress in the steel of 120 MN/'m? or in the bronze of 7) MN/m?. @ Compute the load carried by the bronze without exceeding a stress in the steel of 120 MN/‘m? or in the bronze of 70 MN/m?. ® What is the maximum load P that can be applied without exceeding a stress in steel of 120.MN/m? or in the bronze of 70 MN/m¢. Solution: © Load carried by thestee!. § 255 §,=045, (=),-(@) EJ,-\ES, 5,0) _ 045, 0) 200 “83 5, = 0.6435, When S, = 70 MPa $= 0.643(70) 5, = 44.98 MPa < 120 MPa Po=AsS, = (500)(44.98) P;= 40481 N ® Load carried by the bronze. Py= ApS @ Max. lead P. EM,=0 PYG) =P, (2)+ Py (5). GP = 4048112) + 21,000(5) P= 30,998 N 96 [Sante STM) ae Shown in the figure is a section through albalcony. The total uniferm load of 600 KN is supported by three reds of the same area and tnaterial. Assume the floor to be rigid but noté that it does not necessarily remain horizontal. © Compute the load carried by rod C. © Compute the load carried by rod B. & Compute the load carried by rod A. Solution: ® Load carried by rod: C. Safe a Beek §,-8.= dy -3 8, 5,=38,-28 Bq (5) _3Fp(6)_ 2P, (6) AE” AE ~ AE 5P,=18Py-1P. 5P,=18Py-24P, Py =36P5-24P. IM, =0 Pp (4) + P. (6) = (6003) Py=450-15P. Py +Py+P,=600 (26 P_-24P)+P3 +P, = 600 4.6 Pp-14P, = 600 4.6 (450-15 P.)-14 P= 600 2070 - 6.9P,-1.4P,= 600 Pes A771 kN ® Load carried by rod B Py =450- 1.57. Py = 450 - 1.5(177.11) m Pp= 184.34 kN ; @ Load carried by rod A. Py =3.6Pp-24Pe Pa = 3.6(184.34) - 2.6(177.11) P, = 238.56 kN ee ie Three rods, each with an area of 300 mm2, jointly support the load af 20 kN, as shown. Assuming there was no slack or stress in the rods before the load was applied, find the stress in each rod. Here, E, = 200 x 10° N/m? and B, = 83 x 10° N/m? Solution: Cos 20" = b Ly= 3464 m 6, = 5, Cos 30° Ps SL _ (St : (@),-(@)2 ol Sy (3.464) _ S, Cos 30" (3) bn 6 200 D 8s 5, = 0.31126 5, a 1OkN P,+2P, Cos 30° = 10 kN Sy Ag + 2.Ay S, Cos 30° = 10(1000) 5, (300) + 2(300){0.31126 $,) Cos 30° = 10,000 S,= 21.66 MPa Sp = 0.31126(21.66) Sy = 6.74 MPa Le sTuAi a ‘Three bars, AB, AC, and AD, are pinned together to support a load P = 20 KN as shown. Horizontal movement is prevented at joint A by the short horizontal strut AE. For the steel bar, A = 200 mum? and E = 200 GPa. For each aluminum bar, A =400 mm? and E=70GPa © Compute the stress of member AC. ® Compute the stress of member AB. ® ‘Compute the stress of member AD. @ Compute the force in the strut AE. Solution: © Stress of member AC. San= Bac Cos 30° BL PL (Zt) a” (Rt), Paap (3/Cos 30") — Pac Cos 30" (3) (00) 70) (200)(200) Pag= 0.525 Pac Byp= Sac Cos 45° PL PL 5 (Fe) ." Fe), Pap G/Cos 45") Pac (3) Cos 45 = SOT is (00)(200) Pap= 0.35 Pac Pac + Pap Cos 45" + Pag Cos30" = 20 Pac + (0.35 P,c)\Cos 45° + (0.525 Pye) Cos 30" =20 Pac= 1175 kN Pag = (0.525)(11.75) = 6.169 kh Pap = (0.35)(11.75) = 4.113 kN pen P S= A Sac =12-75{1000) 4.73 (1000 = 58.75 MPa ® Stress.of member AB. Sas ~fisittoon) = 15.42 MPa ® Stressof member AD. oie ~A3co00) = 10.28 MPa © Force in the strut AE. EH=0 Par + Pa Sin 45°=Pap Sin 30" Pre + 4.114 Sin 45° = 6.169 Sin 30° Pag = 0.176 kN Pape 176.N Three bars, AB, AC and AD ace pinned together to support.a load P as shown in the figure, Horizontal movement is prevented at joint.A by the short horizontal ateut AF Determine the maximum value of P that will not exceed an aluminum stress of 40 MPa Or 8 Steel stress of 120 MPa, For steel bar A = 200 mm? and far each aluminum bar, A= 400 mm. Solution: SL batt BS 5 3 a Pap =~ Ab. TAD. i; . bxc= Cas 30"~ Cos 45° s ‘Sac @)_ San (8/Cos 30°) _ Sap @/Cos 45") hs 200 = Cos 30" -*4 Cosas oe g Sac =381 S,p2 5.71 Sap P when Sac = 120 MPa San= eet = 31.5 MPa <40 MPa Sqn=220 = 71.02 MPa < 40MPa Therefore, use Sac = 120 MPa San = 315 MPa Sap= 21.02 MPa P= Pay Cos 20" + Pac + Pap Cos a5" Pan= Sap Ang Pac= Sac Ane Pan =Sap Aap P- (21.5)(400) Cos 30° + (120)(200) + (21,02400) Cos 45° P = 40.856 MPa P= 40.856 KN EA A steel red with a cross-sectional area of 150 mm? is stretched between Iwo fixed points. The tensile load at 20°C is 5000 N. @ What will be the stress at - 20°C? @ At what temperature will the stress be zero? Assume = 11.7 mm/em’ C} and E = 200 x 10° Ném?, Solution: L @ b=5,+8 i SL PLL - nolaT+ ste Py S00 oe ey40" 5000 200 x 198 = 827% 10H) + Fer 00 x 108) $= 127 MPs ef-—F = @ i085; & Oy cat RE or . 5000, [em ie = 20") = fi (7 ONT - 20) = Te6 105 Ta T=342°C A steel rod is stretched between two rigid walls and carries a tensile load of S000 N. at 20°C. If the allowable stress is not to exceed 130 MN/m? at - 20°C, what is the minimum diameter of the rod? Assume a= 11.7 um/(m'C) and E= 200 GPa. Salution: Ie 10 a 0. 200 x 108 = 117 x O40} oa TOR) y sae A= 1074 met sae Fe? = 1574 mm? ya ds 13.22 mm Steel railroad rails 10 m long are laid with a clearance of 3 mm at a temperature of 15°C. At what temperature will the rails just touch? What stress jwill be induced in the rails at that temperature if there were no initial clearance? Assume om 11,7 x 10% m/(m'C) and E = 200GPa. Solution: 8, =0L. aT 3 = (11.7 x 10%}(10,000)T - 15) 10m- 3mm T=4968C aa Ata temperature of 90°C, a steel tire 10 mm thick and 75 mm wide that is to be shrunk onto a locomotive driving wheel 1.8 m. in diameter just fits over the wheel, which is at a temperature of 20°C. Determine the contact pressure between the-tire and the wheel after the assembly cools to 20°C. Neglect the deformation of the wheel caused by the pressure of the tire, Assume o = 117 um/{m'C) and E = 200 10° N/m? Solution: hr I 5-8, fe otis FL. Lat feast | sy os 7 ? i Fs by90 - P5000 xo ET EO ‘MPO - 20) P=122890N 2P = F=pDL 2(122,850) = P(1800)(75) P =1.82 MPa ED: AU130°C, a bronze hoop 20 mm thick whose inside diameter iri 600 mm just fits snugly over a steel hoop 15 mm thick. Both hoops are 100 mm wide. Compute the contact pressure between the boops when the temperature drops to 20°C, Neglect the possibility that the inner ring may buckle. For steel, E = 200 GPa and - @eTL7um/(m’C). For bronze, E = 83 GPa and o = 19. umy/im'C), Solution: 2Pg=2P, : Pye P=P Sy By = By + 8, = Fok ata, Be (al AT, - RE-= (aL aT. Ree (19: 10°99(130 - 20) ~ poe = 011.7 x 108)(130- 20) + TOO(2O)(es000) i 803 x 10% = 602 x10? P + 3.33x109P 803 x 10%=9.35 x 109P P = 85882 N 2P = pDL 2(85882) = p(600)(100) p=286MPa

Você também pode gostar